Logo da Teachy
Entrar

Questão sobre Inequação Logarítmica

Fonte: IME-2FASE


Matemática

Inequação Logarítmica

Médio

(IME-2FASE 2015) - Questão Médio de Matemática


Gabarito

Para resolver a inequação $\frac{4}{\log _{3} x^{2}-2}+\log _{x} \frac{1}{9}>1$, devemos primeiro determinar o domínio dos valores de $x$ para os quais a inequação é definida. **1. Condições de Existência:** * Para $\log_3 x^2$: A base do logaritmo ($3$) é positiva e diferente de $1$. O argumento $x^2$ deve ser positivo, ou seja, $x^2 > 0 \Rightarrow x \neq 0$. * Para $\log_x \frac{1}{9}$: A base $x$ deve ser positiva e diferente de $1$, ou seja, $x > 0$ e $x \neq 1$. O argumento $\frac{1}{9}$ é positivo. * Para o denominador $\log_3 x^2 - 2 \neq 0$: $\log_3 x^2 \neq 2$ $x^2 \neq 3^2$ $x^2 \neq 9$ $x \neq \pm 3$. Combinando todas as condições, o domínio da inequação é $x \in \mathbb{R}$ tal que $x > 0$, $x \neq 1$, $x \neq 3$. **2. Simplificação da Inequação:** * Podemos reescrever $\log_3 x^2$ como $2 \log_3 |x|$. Como $x > 0$ no nosso domínio, temos $2 \log_3 x$. * Podemos reescrever $\log_x \frac{1}{9}$ usando a mudança de base: $\log_x \frac{1}{9} = \frac{\log_3 (1/9)}{\log_3 x} = \frac{\log_3 3^{-2}}{\log_3 x} = \frac{-2}{\log_3 x}$. Substituindo estas simplificações na inequação original: $\frac{4}{2 \log_3 x - 2} - \frac{2}{\log_3 x} > 1$ $\frac{4}{2(\log_3 x - 1)} - \frac{2}{\log_3 x} > 1$ $\frac{2}{\log_3 x - 1} - \frac{2}{\log_3 x} > 1$ **3. Substituição de Variável:** Seja $y = \log_3 x$. A inequação torna-se: $\frac{2}{y-1} - \frac{2}{y} > 1$ **4. Resolução da Inequação em $y$:** $\frac{2y - 2(y-1)}{y(y-1)} > 1$ $\frac{2y - 2y + 2}{y(y-1)} > 1$ $\frac{2}{y(y-1)} > 1$ Agora, trazemos o $1$ para o lado esquerdo e unificamos os termos: $\frac{2}{y(y-1)} - 1 > 0$ $\frac{2 - y(y-1)}{y(y-1)} > 0$ $\frac{2 - y^2 + y}{y(y-1)} > 0$ $\frac{-y^2 + y + 2}{y(y-1)} > 0$ Para facilitar a análise de sinal, multiplicamos o numerador e o denominador por $-1$ (o que inverte o sinal do numerador e mantém o denominador, ou simplesmente multiplicamos a fração por $-1$ e invertemos o sinal da inequação): $\frac{y^2 - y - 2}{y(y-1)} < 0$ Fatoramos o numerador: $y^2 - y - 2 = (y-2)(y+1)$. Então, a inequação fica: $\frac{(y-2)(y+1)}{y(y-1)} < 0$ Os pontos críticos para $y$ (onde o numerador ou o denominador é zero) são $y = -1, y = 0, y = 1, y = 2$. Vamos analisar o sinal da expressão nos intervalos determinados por esses pontos: | Intervalo | $y+1$ | $y$ | $y-1$ | $y-2$ | Numerador $(y+1)(y-2)$ | Denominador $y(y-1)$ | Fração | | :-------------- | :---- | :---- | :---- | :---- | :--------------------- | :------------------- | :----- | | $y < -1$ | $-$ | $-$ | $-$ | $-$ | $+$ | $+$ | $+$ | | $-1 < y < 0$ | $+$ | $-$ | $-$ | $-$ | $-$ | $+$ | $-$ | | $0 < y < 1$ | $+$ | $+$ | $-$ | $-$ | $-$ | $-$ | $+$ | | $1 < y < 2$ | $+$ | $+$ | $+$ | $-$ | $-$ | $+$ | $-$ | | $y > 2$ | $+$ | $+$ | $+$ | $+$ | $+$ | $+$ | $+$ | Queremos que a fração seja menor que zero (negativa). Portanto, os intervalos para $y$ são: $-1 < y < 0$ ou $1 < y < 2$. **5. Retornando à variável $x$:** Lembramos que $y = \log_3 x$. **Caso 1:** $-1 < \log_3 x < 0$ Como $3^t$ é uma função crescente, aplicamos $3^{(\cdot)}$ a todos os termos: $3^{-1} < x < 3^0$ $\frac{1}{3} < x < 1$ **Caso 2:** $1 < \log_3 x < 2$ Aplicamos $3^{(\cdot)}$ a todos os termos: $3^1 < x < 3^2$ $3 < x < 9$ **6. Verificação com o Domínio:** O domínio estabelecido no início foi $x > 0$, $x \neq 1$, $x \neq 3$. * Para $\frac{1}{3} < x < 1$: Todos os valores estão no domínio. * Para $3 < x < 9$: Todos os valores estão no domínio. Assim, os valores reais de $x$ que satisfazem a inequação são $x \in \left(\frac{1}{3}, 1\right) \cup (3, 9)$. The final answer is $\boxed{\left(\frac{1}{3}, 1\right) \cup (3, 9)}$.

Iara Tip

Está montando uma prova ou lista de exercícios?

Na plataforma da Teachy é possível gerar esses materiais automaticamente, sem perder horas buscando por questões 😉

2025 - Todos os direitos reservados

Termos de usoAviso de PrivacidadeAviso de Cookies